Deriving commutator for angular momentum components

I'll try to be more careful with the subscripts. So in the previous post the summations should be like this:##(x_a p_b - x_b p_a) = \left\{\begin{array}{cc}0,&\mbox{ if } a=b \\L_3,&\mbox{ if } (a,b) =(1,2) \\ -L_3,&\mbox{ if } (a,b) =(2,1)\\ L_2,&\mbox{ if } (a,b) =(1,3)\\ -L_2,&\mbox{ if } (a,b) =(3,1) \\ L_1,&\mbox{
  • #1
mr_sparxx
29
4

Homework Statement



Prove that

## [L_a,L_b] = i \hbar \epsilon_{abc} L_c ##

using Einstein summation convention.

I think I have achieved the solution but I am not sure of my last steps, since this is one of my first excersises using this convention.

Homework Equations


[/B]
## (1) [L_a,p_b] = i \hbar \epsilon_{abc} p_c ##
## (2) [L_a,x_b] = i \hbar \epsilon_{abc} x_c ##
## (3) L_a = \epsilon_{abc} x_b p_c ##
## (4) [AB,C] = A [B,C] + [A,C] B ##
## (5) \epsilon_{abc} = \epsilon_{bca} = \epsilon_{cab} ##
## (6) \epsilon_{abc} \epsilon_{auv} = \delta_{bu} \delta_{cv}- \delta_{bv} \delta_{cu}##

3. The Attempt at a Solution

Well, using (3) and (4)

## [L_a,L_b] = [\epsilon_{auv} x_u p_v, L_b] = \epsilon_{auv} x_u [p_v, L_b] + \epsilon_{auv} [x_u , L_b] p_v ##

using (1) and (2), I get:

## \epsilon_{auv} x_u (- i \hbar \epsilon_{bvw}p_w)+\epsilon_{auv} x_u ((- i \hbar \epsilon_{but} x_t) p_v = - i \hbar (\epsilon_{auv} \epsilon_{bvw} x_u p_w + \epsilon_{auv} \epsilon_{but} x_t p_v )##

And using (5) and (6) :

## - i \hbar (\epsilon_{vau} \epsilon_{vwb} x_u p_w + \epsilon_{uva} \epsilon_{utb} x_t p_v ) = - i \hbar (x_b p_a - x_u x_u + x_t x_t - x_a p_b) ##

This expression reduces to

## i \hbar (x_a p_b - x_b p_a) ##

I see this is the expression for the "c" component of the angular momentum but I would like to get the final expression using Einstein's notation and the Levi-Civita, so I guess this could be written as

## i \hbar \epsilon_{cab} x_a p_b ##

Using (3) I get :

## i \hbar L_c ##

Which is different to what I want to prove.

## [L_a,L_b] = i \hbar \epsilon_{abc} L_c ##

Then my assumption is that the Levi Civita symbol in the original equation makes explicit that c is different from a and b and that

## [L_b,L_a] = - [L_a,L_b] ##

So would it be correct to do the following in order to arrive to the solution?

## i \hbar \epsilon_{cab} x_a p_b = i \hbar \epsilon_{abc} \epsilon_{cab} x_a p_b = i \hbar \epsilon_{abc} L_c##
 
Physics news on Phys.org
  • #2
mr_sparxx said:
## i \hbar (x_a p_b - x_b p_a) ##

... I guess this could be written as

## i \hbar \epsilon_{cab} x_a p_b ##

These two expressions are not equivalent. The first expression makes no reference to index c, whereas the second expression does explicitly reference c. Also, the first expression contains fixed values of a and b. But the second expression is a summation over all values of a and b.

I will let you think about this a bit more.
 
  • #3
I knew I was guessing... :eek:
Thanks for pointing that out!

Now I am very uncomfortable with the previous steps (and with this notation in general, for now at least).:)

I was trying to avoid summation symbols but I think I have to. When I wrote

mr_sparxx said:
## - i \hbar (\epsilon_{vau} \epsilon_{vwb} x_u p_w + \epsilon_{uva} \epsilon_{utb} x_t p_v ) = - i \hbar (x_b p_a - x_u x_u + x_t x_t - x_a p_b) ##

This expression reduces to

## i \hbar (x_a p_b - x_b p_a) ##
Am I really doing
## - i \hbar ( \sum_u \sum_v \sum_w \epsilon_{vau} \epsilon_{vwb} x_u p_w + \sum_u \sum_v \sum_t \epsilon_{uva} \epsilon_{utb} x_t p_v ) = \\ =- i \hbar ( \sum_u \sum_v \sum_w (\delta_{aw}\delta_{ub}-\delta_{ab}\delta_{uw})x_u p_w + \sum_u \sum_v \sum_t (\delta_{vt}\delta_{ab}-\delta_{vb}\delta_{at}) x_t p_v ) = \\ = - i \hbar ( \sum_v x_b p_a - \sum_v \sum_u x_u p_u + \sum_u \sum_v x_v p_v - \sum_u x_a p_b) = - i \hbar ( \sum_v x_b p_a- \sum_u x_a p_b)= - i \hbar (3 x_b p_a- 3 x_a p_b) ##

?
 
  • #4
mr_sparxx said:
Am I really doing
## - i \hbar ( \sum_u \sum_v \sum_w \epsilon_{vau} \epsilon_{vwb} x_u p_w + \sum_u \sum_v \sum_t \epsilon_{uva} \epsilon_{utb} x_t p_v ) = \\ =- i \hbar ( \sum_u \sum_v \sum_w (\delta_{aw}\delta_{ub}-\delta_{ab}\delta_{uw})x_u p_w + \sum_u \sum_v \sum_t (\delta_{vt}\delta_{ab}-\delta_{vb}\delta_{at}) x_t p_v )##

?

In going from ##\sum_u \sum_v \sum_w \epsilon_{vau} \epsilon_{vwb} x_u p_w ## to ##\sum_u \sum_v \sum_w (\delta_{aw}\delta_{ub}-\delta_{ab}\delta_{uw})x_u p_w ##, should there still be a sum over ##v##?

Note, your original work looked good to me all the way to the result of ##i \hbar (x_a p_b- x_b p_a) ## (except for a couple of typos where you had ##x_u x_u## instead of ##x_u p_u## and ##x_t x_t## instead of ##x_t p_t##). The problem was when you said that ##i \hbar (x_a p_b- x_b p_a) = i\hbar \epsilon_{cab} x_a p_b ##.
 
  • Like
Likes mr_sparxx
  • #5
True. I was a bit in panic. Those 3's made no sense at all. I'll think a bit more about it.
 
  • #6
mr_sparxx said:
This expression reduces to

## i \hbar (x_a p_b - x_b p_a) ##

Well I think I've got it by exploring all possible values:

##(x_a p_b - x_b p_a) = \left\{\begin{array}{cc}0,&\mbox{ if } a=b \\L_3,&\mbox{ if } (a,b) =(1,2) \\ -L_3,&\mbox{ if } (a,b) =(2,1)\\ L_2,&\mbox{ if } (a,b) =(1,3)\\ -L_2,&\mbox{ if } (a,b) =(3,1) \\ L_1,&\mbox{ if } (a,b) =(2,3) \\ -L_1,&\mbox{ if } (a,b) =(3,2) \end{array}\right . = \epsilon_{abc} L_c##

However, I still wonder if I have done it correctly, and if I have, if there is a more elegant algebraic solution for this (with no numbers).

Moreover, as I think about it I am not sure if I even understand the original identity:

## [L_a,L_b] = i \hbar \epsilon_{abc} L_c ##

Is here a summation over all possible values of c but none over a and b because they are fixed on the left hand side?

My apologies but as I said before this is the first time I use Einstein's convention.
 
  • #7
mr_sparxx said:
However, I still wonder if I have done it correctly, and if I have, if there is a more elegant algebraic solution for this (with no numbers).
I think it is correct. You just have to realize that ##x_a p_b - x_b p_b## is equivalent to ##\epsilon_{abc} L_c##.

Moreover, as I think about it I am not sure if I even understand the original identity:

## [L_a,L_b] = i \hbar \epsilon_{abc} L_c ##

Is here a summation over all possible values of c but none over a and b because they are fixed on the left hand side?

Yes, that's right. For example ## [L_1,L_2] = i \hbar \epsilon_{12c} L_c ## where c is summed over the values 1, 2, 3. So, the right hand side reduces to## i \hbar L_3##.
 
  • Like
Likes mr_sparxx
  • #8
I think I understand Einstein's convention a little more. I will keep working on this topic: ## [L_i, L^2] = 0## next. I hope I manage to prove this alone.

Thanks for your help!
 
  • #9
mr_sparxx said:
I will keep working on this topic: ## [L_i, L^2] = 0## next. I hope I manage to prove this alone.

Thanks for your help!
I can't get this one correct either... I'll post a new thread anyway. Thanks a lot!
 

Related to Deriving commutator for angular momentum components

1. What is a commutator for angular momentum components?

A commutator for angular momentum components is a mathematical operator that describes how two angular momentum components behave when they are applied in different orders. It is used to determine the uncertainty in measuring two angular momentum components simultaneously.

2. How is the commutator for angular momentum components derived?

The commutator for angular momentum components is derived using the fundamental commutation relations for angular momentum operators. These relations involve the position and momentum operators, and can be used to derive the commutator for any two angular momentum components.

3. Why is the commutator for angular momentum components important?

The commutator for angular momentum components is important because it allows us to understand the relationship between different angular momentum components and how they affect each other. It also plays a crucial role in quantum mechanics and is used in calculations for various physical systems.

4. What is the physical interpretation of the commutator for angular momentum components?

The physical interpretation of the commutator for angular momentum components is that it represents the uncertainty in measuring two angular momentum components simultaneously. It tells us how much the measurement of one component affects the measurement of the other, and vice versa.

5. Are there any applications of the commutator for angular momentum components?

Yes, there are several applications of the commutator for angular momentum components in physics. It is used in quantum mechanics to study the behavior of particles and systems, and it is also used in the study of rotational motion and spin in classical mechanics. It also has applications in fields such as chemistry, spectroscopy, and astronomy.

Similar threads

  • Advanced Physics Homework Help
Replies
3
Views
1K
  • Advanced Physics Homework Help
Replies
2
Views
5K
  • Advanced Physics Homework Help
Replies
7
Views
2K
  • Advanced Physics Homework Help
Replies
2
Views
4K
  • Advanced Physics Homework Help
Replies
4
Views
7K
  • Introductory Physics Homework Help
Replies
5
Views
1K
  • Advanced Physics Homework Help
Replies
2
Views
2K
  • Advanced Physics Homework Help
Replies
26
Views
3K
  • Advanced Physics Homework Help
Replies
4
Views
1K
Replies
7
Views
2K
Back
Top